2018in ServiceExam

You might also like

Download as pdf or txt
Download as pdf or txt
You are on page 1of 74

2018 IN-SERVICE EXAMINATION

2018 ACOFP IN-SERVICE EXAMINATION

1. A 63-year-old plumber presents for follow-up of recently diagnosed type 2 diabetes mellitus. He has a
strong family history of coronary artery disease. His father died from a myocardial infarction at age 48.
Physical examination is normal and blood pressure is 140/90 mmHg. Laboratory studies reveal a BUN of
15 mg/dL and a creatinine of 0.9 mg/dL. The current recommendation regarding urine screening in this
patient is to provide

A. no testing unless renal insufficiency develops


B. annual testing with standard urine dipstick beginning five years after diagnosis
C. annual testing with standard urine dipstick beginning at the time of diagnosis
D. annual testing for microalbuminuria beginning at the time of diagnosis
E. annual testing for microalbuminuria beginning five years after diagnosis

2. The main reason for giving iodinated contrast prior to a CT scan of the head is to increase the likelihood
of visualizing a/an

A. intracranial neoplasm
B. acute thrombotic stroke
C. intracranial hemorrhage
D. epidural hematoma
E. subdural hematoma

3. A female patient presents for evaluation of suspected asthma. She reports coughing and wheezing that
occurs 2 times each week. Similar symptoms occur each year in the spring. Her symptoms resolve
completely with the use of inhaled albuterol (Ventolin®). She has not needed to change any of her
normal activities and she denies any symptoms at night. Her lungs at this time are clear to auscultation,
and office pulmonary function tests reveal normal FEV1. The most likely diagnosis is

A. mild intermittent asthma


B. mild persistent asthma
C. moderate intermittent asthma
D. moderate persistent asthma
E. chronic bronchitis

4
2018 ACOFP IN-SERVICE EXAMINATION

4. A spry 84-year-old male with hypertension controlled by a low-dose diuretic presents with his grandson.
The grandson informs you that the elderly gentleman has fallen several times during the previous week.
On questioning the patient denies having fallen, stating that he just lost his balance and slid to the floor.
Vital signs are stable and physical examination is unremarkable aside from some bruising to the left hip
and forearm. There is no significant bony tenderness. Which of the following is the most appropriate
course of action?

A. ECG and chemistry profile


B. lumbosacral spine and pelvic radiographs
C. prescription for a front-wheeled walker
D. referral for aquatherapy and gait training
E. referral to adult protective services to investigate possible abuse

5. A 72-year-old male presents for his annual health maintenance examination. He reports no complaints.
His wife states that she thinks he is becoming hard of hearing. Office audiometry reveals markedly
reduced bilateral hearing in the 6000-8000 Hz range, with only slightly diminished hearing at lower
decibels. This is most representative of

A. drug-induced ototoxicity
B. neoplasm
C. otosclerosis
D. presbycusis
E. cerumen impaction

6. Which of the following statements is correct regarding patients going through alcohol withdrawal?

A. they are at risk for hyperglycemia


B. they will develop delirium tremens
C. they should always receive thiamine and a multivitamin
D. propranolol can decrease the likelihood of withdrawal seizures and hallucinations
E. they should receive magnesium sulfate for seizure prophylaxis

5
2018 ACOFP IN-SERVICE EXAMINATION

7. Two hours after returning from a picnic, a patient develops intense abdominal cramping followed by
explosive, non-bloody diarrhea and vomiting. The most likely cause of these symptoms is

A. Campylobacter
B. Clostridium
C. Salmonella
D. Shigella
E. Staphylococcus

8. A 38-year-old weekend athlete presents to the office complaining of shoulder pain focused over the
lateral aspect of his right shoulder, which is frequently a problem at night. On examination you find more
pain on active abduction than passive abduction; pain is observed when the shoulder is positioned at
90 degrees flexion, 45 degrees abduction, and maximal internal rotation, and a loading force is applied
caudad. The most likely diagnosis is

A. acromioclavicular subluxation
B. infraspinatus tear
C. glenoid tear
D. supraspinatus tear
E. subdeltoid bursitis

9. A 55-year-old male presents to the office with the complaint of symmetrical bilateral finger and hand
numbness. When his hands get cold, they are painful and change color from white to blue to red. If he
keeps his hands warm, he has no problems. The most appropriate treatment is

A. ACE inhibitor
B. benzodiazepine
C. calcium channel blocker
D. thiazide diuretic
E. thyroid hormone

6
2018 ACOFP IN-SERVICE EXAMINATION

10. A 30-year-old male presents for evaluation of left hip pain. Hip range of motion is normal. The left ASIS
is superior and the left PSIS is inferior. A standing flexion test reveals the left PSIS to be more superior
when the patient is fully flexed forward. A seated flexion test is positive on the right. The most likely
diagnosis is

A. left innominate anterior


B. left innominate posterior
C. right innominate anterior
D. right innominate posterior
E. sacral torsion

11. A 22-year-old female is evaluated for an annual health maintenance examination. She is sexually
active, consistently utilizes oral contraceptives, and is in a stable monogamous relationship. Her
vaccination history reveals that she received an MMR vaccination at 13 months of age and that she had
chickenpox at age 6, but she has not received the varicella vaccine. Which of the following
combinations of preventive services is most appropriate for this patient?

A. Pap smear and MMR


B. Pap smear and varicella vaccine
C. Pap smear, MMR, and chlamydia screening
D. Pap smear, MMR, and varicella titer
E. Pap smear, MMR, and varicella vaccine

12. A 50-year-old female with a history of untreated hyperthyroidism suffers a compression fracture of the
L4 vertebra. Which of the following is an urgent indication for laminectomy?

A. a lower extremity neurologic deficit such as weakness of the foot


B. acute urinary incontinence
C. chronic low back pain
D. depression from chronic pain
E. herniated disk observed on MRI

7
2018 ACOFP IN-SERVICE EXAMINATION

13. Which of the following is correct regarding current Centers for Disease Control and Prevention
recommendations for hepatitis B screening in pregnancy?

A. all pregnant women should be screened for hepatitis B surface antigen


B. only pregnant women with high risk histories should be screened for hepatitis B surface antigen
C. all pregnant women should be screened with the anti-hepatitis B core antibody test
D. the Centers for Disease Control and Prevention does not currently recommend screening pregnant
women for hepatitis B status
E. only pregnant women who have immigrated from Asian countries need to be screened for hepatitis B

14. A 40-year-old female has a Pap smear finding of atypical glandular cells. The most appropriate next
step is

A. loop electrosurgical excision procedure (LEEP)


B. perform a colposcopy and endocervical curettage
C. refer the patient to a gynecologic oncologist
D. repeat the Pap smear in six months
E. repeat the Pap smear in three months utilizing thin prep methodology

15. What are the current recommendations of the U.S. Preventive Services Task Force (USPSTF)
regarding screening for osteoporosis with bone mineral density testing?

A. only high risk women should be screened


B. there is strong evidence against screening
C. all post-menopausal women should be screened
D. all women over the age of 65 should be screened
E. evidence is insufficient to recommend for or against screening

8
2018 ACOFP IN-SERVICE EXAMINATION

16. A 14-year-old female with a history of mild persistent asthma presents with increasing amounts of
coughing and dyspnea during exercise. Previously her asthma had been well controlled with
montelukast (Singulair®) and albuterol (Proventil®) as needed. She is now using her albuterol
(Proventil®) approximately three times daily and occasionally at night. She denies any known exposure
to environmental allergies, cigarette smoke, or gastroesophageal reflux disease symptoms. Her
physical examination is unremarkable. The most appropriate next step in management is

A. azithromycin (Zithromax®)
B. fluticasone (Flovent®)
C. ipratropium (Atrovent®)
D. loratadine (Claritin®)
E. methacholine challenge

17. A patient presents with the complaint of chronic heartburn and epigastric discomfort. In the process of
performing the osteopathic examination, you note a tender point posteriorly. At which level would you
expect to find this Chapman reflex point?

A. C3
B. L1
C. T10
D. T12
E. T5

18. Which of the following factors allows you to code at a higher complexity visit based on counseling or
coordination of care?

A. complexity of care
B. presence of a psychiatric condition complicating care
C. number of tests ordered
D. number of referrals made
E. time

9
2018 ACOFP IN-SERVICE EXAMINATION

19. The arterial supply to the frontal and maxillary sinuses may be improved by treating somatic dysfunction
at

A. T1-T4
B. T5-T9
C. T10-T11
D. T12-L2
E. C3-C5

20. A 28-year-old male presents for evaluation of a rash, as shown in exhibit 1 and exhibit 2. He reports
that he initially noticed an irritated sensation and then developed the lesions. Physical examination is
otherwise normal. The most likely diagnosis is

10
2018 ACOFP IN-SERVICE EXAMINATION

A. contact dermatitis
B. impetigo
C. pityriasis rosea
D. tinea corporis
E. varicella zoster

21. At what time during pregnancy are group B streptococci cultures recommended?

A. 10-12 weeks
B. 18-20 weeks
C. 26-28 weeks
D. 35-37 weeks
E. at the onset of labor

11
2018 ACOFP IN-SERVICE EXAMINATION

22. A 24-year-old female is evaluated for neck pain. Examination reveals restriction to right rotation with
normal sidebending bilaterally. These findings most likely indicate somatic dysfunction at which of the
following levels?

A. occiput-C1
B. C1-C2
C. C2-C3
D. C3-C4
E. C4-C5

23. A 17-year-old male with chronic persistent asthma presents for evaluation. History reveals that he is
compliant with his medications and has experienced reduced episodes of wheezing. He does note that
when he exerts himself or becomes upset, wheezing occurs. In order to reduce the effect of
sympathetic tone, to which of the following anatomic structures should osteopathic manipulative
treatment be performed?

A. C1-C3
B. T1-T6
C. T8-T12
D. costal nerves
E. phrenic nerve

24. Which of the following is most likely to place a patient at increased risk for a wound infection following a
ventral incisional hernia repair?

A. female gender
B. male gender
C. multiple sex partners
D. smoking
E. young age

12
2018 ACOFP IN-SERVICE EXAMINATION

25. A 32-year-old female presents 45 minutes after cutting her left index finger in her kitchen with a clean
knife. The wound appears clean. Following examination you close the wound with simple interrupted
sutures. She reports that she is current on all childhood immunizations; however, her last tetanus shot
was 14 years ago. The most appropriate course of action is to administer which of the following
vaccines?

A. diphtheria, tetanus, pertussis (DTaP)


B. tetanus and diphtheria (Td)
C. tetanus and diphtheria (Td) and tetanus immune globulin (TIG)
D. tetanus immune globulin (TIG)
E. tetanus, diphtheria, pertussis (Tdap)

26. A 24-year-old female is diagnosed with rheumatoid arthritis. According to the American College of
Rheumatology, the class of medication that should be used as early as possible in order to control
symptoms and delay disease progression is

A. antibiotics
B. disease-modifying antirheumatologic drugs
C. narcotics
D. nonsteroidal antiinflammatory drugs
E. steroids

27. Which of the following tests is most specific for evaluating the extent of hepatocellular damage in a
patient with chronic hepatitis C?

A. anti-hepatitis C antibodies (qualitative)


B. elevated erythrocyte sedimentation rate
C. elevated liver enzymes
D. hepatic biopsy
E. hepatitis C viral load (quantitative) (HCV by PCR)

13
2018 ACOFP IN-SERVICE EXAMINATION

28. During mid-season a 17-year-old high school track athlete develops pain in his right foot. Onset during
practice was sudden and involved no trauma. He reports that it is painless at rest and on awakening,
but exacerbates with running. Walking is not uncomfortable. Arch taping helps but does not eliminate
the pain. Examination reveals that it is centered in his midfoot. There is no erythema or edema. The
most likely diagnosis is

A. cuneiform torsion
B. Lisfranc dislocation
C. pes planus
D. plantar fasciitis
E. stress fracture

29. An 80-year-old female presents with all of her medications following hospitalization for acute cardiac
decompensation. She admits to having no appetite and occasional nausea. You note that she now has
a fine resting tremor. ECG is obtained, as shown in the exhibit. The most likely diagnosis is

A. acute anterior wall myocardial infarction


B. acute inferior wall myocardial infarction
C. digitalis toxicity
D. hyperkalemia
E. pericarditis

14
2018 ACOFP IN-SERVICE EXAMINATION

30. A Medicare patient presents to a medical office early in the day. Later that day the patient presents to
the hospital and is admitted by the same physician. The visits should be coded by using

A. both the office visit and hospital admission codes


B. both the office visit and hospital admission codes with a -25 modifier
C. the appropriate level hospital admission code
D. the appropriate level office visit code
E. the office visit code but upcode one level

31. A 38-year-old male runner presents with right hip pain. Standing flexion test is positive on the right and
seated flexion test is negative. PSIS is superior on the right and ASIS is inferior on the right. Tightness
in which of the following muscles is most likely to cause this dysfunction?

A. left hamstring
B. left piriformis
C. right iliopsoas
D. right piriformis
E. right quadriceps

32. An 82-year-old active male with a history of T12 compression fracture presents with a two-day history of
new low back pain. He describes the pain as dull, constant, and slightly greater on the right. Quadratus
lumborum tension is palpated on the right. He has restriction of right lumbar sidebending. The
transverse processes of L2-L4 are posterior on the right. This asymmetry does not improve in flexion or
extension. The somatic dysfunction is described as

A. L1 neutral, rotated left, sidebent right


B. L2 flexed, rotated right, sidebent right
C. L2-L4 rotated right, sidebent left
D. L2-L4 rotated right, sidebent right
E. L4 flexed, rotated right, sidebent right

15
2018 ACOFP IN-SERVICE EXAMINATION

33. A 65-year-old male presents for a follow-up evaluation from the emergency department with the
complaint of shoulder pain. He had an MRI last week that revealed a very small tear of the rotator cuff.
He has no acute injury known to him. The pain has been intermittent for 2 years and becomes worse
with impingement maneuvers. He has tried conservative measures at home as well as osteopathic
manipulative treatment to relieve the pain. The most appropriate treatment for this patient is

A. a long arm cast for 4 to 6 weeks


B. a sling and swathe for 1 month
C. high velocity, low amplitude treatment to the cervical spine
D. orthopedic referral for surgical repair
E. subacromial corticosteroid injection

34. A 19-year-old gravida 2 para 0-0-1-0 female at 18 weeks' and 2 days' gestation presents with a 7-day
history of intense pruritus of her hands and armpits. The pruritus is worse at night but lasts throughout
the day. She reports that her boyfriend has similar symptoms. Physical examination reveals a papular,
excoriated rash with burrows on the dorsal surfaces of the hands and in both axillae. The most
appropriate treatment is

A. 1% hydrocortisone (Cetacort®) cream applied twice daily for 1 week


B. 5% permethrin (Lyclear®) cream applied once and rinsed off in 12 hours
C. ivermectin (Stromectol®) 200 mcg/kg single oral dose
D. clotrimazole (Lotrimin®) cream applied twice daily to affected areas
E. prednisone (Deltasone®) 40 mg daily for 3 days

35. A 20-year-old volleyball player is struck on the end of the middle finger by the ball. Examination reveals
tenderness at the dorsum of the distal interphalangeal joint. Passive range of motion is full, but active
range of motion lacks five degrees of terminal extension of the distal interphalangeal joint. Radiographs
are unremarkable. The most appropriate management is

A. early mobilization and hand therapy


B. MRI to further evaluate the extensor tendon
C. referral for immediate surgical repair
D. splint in a ten-degree flexion for 6 to 8 weeks
E. splint in full extension for 6 to 8 weeks

16
2018 ACOFP IN-SERVICE EXAMINATION

36. A 45-year-old male is brought to the emergency department following a motor vehicle collision. History
reveals that he was stopped at a red light when he was hit from behind. He reports that his headrest
was too low, as he never adjusted it properly in his new car. This patient most likely injured the

A. anterior longitudinal ligament


B. interspinous ligament
C. ligamentum flavum
D. posterior longitudinal ligament
E. supraspinous ligament

37. A 65-year-old male presents with a history of smoking and hypertension. Family history is significant for
coronary artery disease. Ultrasound of the abdomen reveals a 4 cm abdominal aortic aneurysm. The
most appropriate next step is

A. MRA of the abdomen


B. MRI of the abdomen
C. noncontrast CT scan of the abdomen
D. observation
E. surgical consultation

38. Which of the following mechanisms is most likely responsible for the increase in β-type natriuretic
peptide seen in congestive heart failure?

A. decreased left ventricular volume


B. increased afterload affecting the kidneys
C. increased preload affecting the kidneys
D. right atrial contraction
E. ventricular myocyte stretch

17
2018 ACOFP IN-SERVICE EXAMINATION

39. A 26-year-old male presents with unremitting back pain at the thoracolumbar junction, which has been
treated repeatedly with physical therapy and osteopathic manipulation with little to no relief. He also
notes loose bowel movements that are often bloody but not painful. He denies traveling, recent
antibiotic use, or fever. The most likely diagnosis is

A. celiac sprue
B. erosive gastritis
C. hemorrhoids
D. infectious colitis
E. ulcerative colitis

40. A 24-year-old female presents for evaluation of a breast mass. Examination reveals a mass in the upper
outer quadrant of the left breast that is mobile, firm, and approximately 1 cm in diameter. No axillary
lymph nodes are palpated. The most likely diagnosis is

A. carcinoma of the breast


B. fibroadenoma
C. intraductal carcinoma
D. mastitis
E. Paget disease of the breast

41. According to the Ottawa ankle rules, which of the following indicates the need for ankle radiographs?

A. patient is able to walk immediately after the injury


B. swelling is present in the ankle joint
C. tenderness is noted at the base of the first toe
D. tenderness is noted at the posterior aspect of the lateral malleolus
E. tenderness is noted distal to the lateral malleolus

18
2018 ACOFP IN-SERVICE EXAMINATION

42. A 48-year-old female had a total hysterectomy due to excessive bleeding from uterine fibroids. She has
no history of abnormal Pap tests, has been in a monogamous relationship since age 20, and has no
history of diethylstilbestrol exposure or any immunocompromised condition. According to the American
Cancer Society Guidelines, how often should this patient have a Pap test?

A. every year
B. every 2 years
C. every 3 years
D. every 5 years
E. Pap tests are not indicated

43. An 82-year-old male with Parkinson disease presents for evaluation of a rash that has been worsening
over the past year. He is noted to have oily scales overlying erythematous patches in his scalp,
especially at the hairline. He also has scaling noted in the eyebrows, external auditory canals, and
nasolabial folds. His only medication is levodopa-carbidopa (Sinemet®). The most appropriate treatment
is

A. a water-based moisturizer
B. fluoridated glucocorticoid cream
C. ketoconazole (Nizoral®) cream
D. metronidazole (Flagyl®) gel
E. topical erythromycin solution

44. A 68-year-old male presents with the complaint of excess thirst and urination, fatigue, and the gradual
onset of impotency over the past six months. His skin has a dark color that also appeared over the past
year. Physical examination is positive for atrophic testes, hepatomegaly, and a firm, freely moveable
thyroid. Laboratory studies reveal a fasting glucose of 274 mg/dL. The most likely diagnosis is

A. Addison disease
B. hemochromatosis
C. pituitary adenoma
D. Sipple syndrome
E. Werner syndrome

19
2018 ACOFP IN-SERVICE EXAMINATION

45. A new patient presents to the office for a health maintenance examination. She reports that she is so
glad she found you because "all the other doctors out there are no good". She compliments your
appearance and states that she is certain you are going to cure her abdominal pain. Caution should be
exercised with this patient as she most likely has

A. borderline personality disorder


B. conversion reaction
C. histrionic personality disorder
D. reaction formation
E. sociopathic personality disorder

46. A 25-year-old male presents to the office with the complaint of right-sided thoracic pain following a fall at
work approximately 1 week ago. He reports that the severity of the pain has decreased throughout the
week, but is still present, especially in maximum inhalation. Ibuprofen (Advil®) 3 times a day seems to
decrease the pain. On examination, ribs 6 to 9 on the right are restricted in inhalation. Radiographs of
the ribs reveal no fracture. An ECG reveals normal sinus rhythm. A rib dysfunction is suspected. Which
of the following is the most likely diagnosis and target for muscle energy treatment?

A. exhalation dysfunction; treatment directed at rib 6


B. exhalation dysfunction; treatment directed at rib 9
C. inhalation dysfunction; treatment directed at rib 6
D. inhalation dysfunction; treatment directed at rib 8
E. inhalation dysfunction; treatment directed at rib 9

47. A healthy 28-year-old male presents to the office after falling backwards against a chair and injuring his
back. He reports that he has pain between his shoulder blades at the level of the inferior angle of the
scapula. Which of the following sets of symptoms would be most consistent with a sympathetic
viscerosomatic response to this patient's injury?

A. belching and dyspepsia


B. difficulty defecating with straining and tenesmus
C. difficulty urinating and pain with intercourse
D. nasal drainage and sneezing
E. pain radiating down the back of the right leg to the back of the knee

20
2018 ACOFP IN-SERVICE EXAMINATION

48. A 15-year-old football player presents to the office following an injury to the right shoulder. History
reveals that immediately following the injury he complained of burning and shooting pain from his neck
into the right upper arm. On examination he complains of muscle weakness of the right deltoid muscle.
A small circular patch on the lateral deltoid that appears to have decreased sensation to touch is noted.
Deep tendon reflexes are +2/4 for biceps, triceps, and brachioradialis bilaterally. The most likely
diagnosis is

A. biceps tendonitis
B. brachial plexus stretch injury
C. deltoid tendonitis
D. impingement syndrome
E. rotator cuff strain

49. A 25-year-old female presents with increased frequency, urgency, and suprapubic tenderness. The
patient has no fever and no known allergies. Urine dipstick reveals positive nitrites and leukocytes. The
most appropriate antibiotic treatment is

A. amoxicillin (Amoxil®) 500 mg 3 times daily for 3 days


B. ciprofloxacin (Cipro®) 500 mg twice daily for 10 days
C. doxycycline (Doryx®) 100 mg twice daily for 7 days
D. metronidazole (Flagyl®) 500 mg twice daily for 7 days
E. trimethoprim-sulfamethoxazole (Bactrim DS®) twice daily for 3 days

50. An 85-year-old female is noted to have a calcium level of 11.8 mg/dL. Which of the following ECG
changes is most likely to occur due to this patient's electrolyte imbalance?

A. third-degree heart block


B. peaked T waves
C. prominent U waves
D. QT interval shortening
E. torsades de pointes

21
2018 ACOFP IN-SERVICE EXAMINATION

51. A 72-year-old male presents for evaluation of a tremor in both his arms that has been progressively
worsening over the last 6 months. The tremor initially began in the left arm, and four months ago began
in the right arm. The tremor only occurs while he is at rest. He reports that he has been moving slower
and has to take much smaller steps. He is taking no new medications. The most appropriate next step
to confirm the diagnosis is

A. complete history and physical examination


B. CT scan of the head with IV contrast
C. lumbar puncture
D. MRI of the brain
E. electroencephalogram

52. A 25-year-old male presents with a 2-hour history of acute shortness of breath and severe right upper
quadrant pain on inspiration. He smokes cigarettes and has a 7 pack-year history. He has no additional
medical history. On examination he is in noticeable distress. Pulse oximetry reveals an oxygen
saturation of 88% on room air. The most likely diagnosis is

A. asthma exacerbation
B. bronchitis
C. pneumonia
D. pulmonary embolism
E. spontaneous pneumothorax

53. The most appropriate first-line prophylactic treatment for exercise-induced asthma is

A. inhaled long-acting β-agonist 15 minutes prior to exercise


B. inhaled short-acting β-agonist 15 minutes prior to exercise
C. inhaled steroids prior to exercise
D. oral leukotriene inhibitors
E. oral steroid therapy

22
2018 ACOFP IN-SERVICE EXAMINATION

54. A 22-year-old female at 35 weeks' gestation presents with a one-day history of right-sided abdominal
pain. History reveals mild preeclampsia for six weeks. She reports that earlier this morning she
experienced two episodes of nausea with vomiting, but has since been able to tolerate liquids.
Overnight she had epistaxis. She is afebrile with an apical heart rate of 104/min and a blood pressure of
126/78 mmHg. Physical examination reveals obvious bilateral epistaxis. Heart and lung examinations
are normal. Abdominal examination reveals right upper quadrant tenderness and uterine enlargement
consistent with her gestational age. There is 1-2+ bilateral lower extremity edema. Laboratory studies
reveal:

Hemoglobin 7.4 g/dL


Leukocyte count 8.6 × 103/mcL
Platelet count 41 × 103/mcL
Aspartate aminotransferase 171 U/L
Alanine aminotransferase 180 U/L
Total bilirubin 4.4 mg/dL
Direct bilirubin 0.8 mg/dL
INR 1.1
Partial thromboplastin time 27 sec
Urinalysis 2+ protein

The most appropriate treatment is

A. acyclovir (Zovirax®)
B. endoscopic retrograde cholangiopancreatography
C. intravenous corticosteroids
D. magnesium sulfate
E. prompt delivery of infant

23
2018 ACOFP IN-SERVICE EXAMINATION

55. A 24-year-old female presents with the complaint of recurrent, pulsating, unilateral headaches. The
headaches last four to six hours, are of moderate to severe intensity, and are accompanied by nausea
and/or photophobia. She reports that her symptoms are aggravated by routine physical activity. This
clinical presentation is most consistent with which of the following types of headache?

A. cluster
B. migraine
C. rebound
D. sinus
E. tension

56. A 25-year-old female presents following a positive home pregnancy test result. She has irregular
menses and would like an accurate date of her gestation. Transvaginal ultrasound reveals a crown-
rump length consistent with an 8 6/7 weeks' gestation, adequate amniotic fluid, and no identifiable
heartbeat. The ultrasound is repeated in two days and there is no change. Examination reveals her
cervix is closed with no bleeding. Quantitative β-HCG levels have begun to decline. The most likely
diagnosis is

A. elective abortion
B. incomplete abortion
C. missed abortion
D. septic abortion
E. threatened abortion

57. A 38-year-old female presents with a 2-week history of right-sided low back pain. The pain is dull and
aching with occasional shooting, burning pain into the thigh and is worse when sitting in the car and
when walking up stairs. She denies trauma. She has normal deep tendon reflexes, normal muscle
strength testing, and negative straight leg testing. Structural examination reveals L2 flexed, rotated
right, sidebent right. Which of the following positions would shorten the most likely involved muscle?

A. extension and internal rotation of the right hip


B. extension of the right hip
C. flexion and external rotation of the right hip
D. lumbar sidebending to the left
E. internal rotation of the right hip

24
2018 ACOFP IN-SERVICE EXAMINATION

58. A decrease in serum aldosterone levels has been demonstrated in hypertensive patients in whom
Chapman tender points were treated at spinal levels

A. occiput, C1-C2
B. C4-C5
C. T2-T3
D. T7-T8
E. T11-T12

59. Where does the sympathetically mediated viscerosomatic reflex from the upper respiratory tract
originate?

A. C1-C2
B. C3-C5
C. C6-C8
D. T1-T4
E. T5-T7

60. In a patient with acute pneumonia, palpable tissue texture abnormalities in the upper thoracic region are
an example of a/an

A. primary thoracic somatic dysfunction


B. somatosomatic reflex
C. somatovisceral reflex
D. viscerosomatic reflex
E. viscerovisceral reflex

25
2018 ACOFP IN-SERVICE EXAMINATION

61. Examination of a 1-week-old neonate reveals cranial dysfunction. Dysfunctional rotation of the occiput
on the atlas (C1) has been shown to be clinically associated with

A. asymmetric head shape


B. fetal alcohol syndrome
C. low birth weight
D. mouth breathing
E. sudden infant death syndrome

62. A 4-year-old male presents for an annual health maintenance examination. Physical examination
reveals eye-related abnormalities. It is noted that the abducens nerve is compromised. This is most
likely to produce

A. astigmatism
B. dry eye
C. hyperopia
D. myopia
E. strabismus

63. A 5-month-old male is brought to the office for evaluation of persistent tearing in his right eye. He is
otherwise well, and all health maintenance examinations have been reported normal. He is current on
immunizations. Physical examination reveals no pathology. The most appropriate course of action is

A. application of cold compresses twice daily until symptoms resolve


B. reassurance that this condition usually resolves by 1 year of age
C. referral to an ophthalmologist for immediate tear duct dilation
D. topical antibiotics to prevent tear duct infection
E. topical corticosteroid drops to facilitate tear duct opening

26
2018 ACOFP IN-SERVICE EXAMINATION

64. A 7-year-old female presents with a one-day history of a lesion on her back. History reveals that it
began as one patch on her back, and has now evolved into multiple, smaller erythematous macules on
her extremities that are itchy and spreading peripherally. She denies any current fevers. Physical
examination reveals the findings shown in exhibit 1 and exhibit 2. The most likely diagnosis is

27
2018 ACOFP IN-SERVICE EXAMINATION

A. erythema infectiosum
B. pityriasis alba
C. pityriasis rosea
D. tinea corporis
E. tinea versicolor

65. The most appropriate initial screening test for an abdominal aortic aneurysm is

A. angiogram
B. CT scan with contrast
C. CT scan without contrast
D. MRI
E. ultrasound

28
2018 ACOFP IN-SERVICE EXAMINATION

66. A 68-year-old male presents to the office with a 1-week history of cough and a temperature of 38.5°C
(101.3°F). Chest radiograph reveals a right middle lobe infiltrate. The most likely cause is

A. Chlamydophila pneumoniae
B. Haemophilus influenzae
C. Mycoplasma pneumoniae
D. Streptococcus pneumoniae
E. Legionella species

67. Which of the following are the components of the female athlete triad?

A. depression, weight loss, sports-related injury


B. low energy available, menstrual dysfunction, altered bone mineral density
C. poor sports performance, low self-esteem, injury
D. sports participation, emotional disturbance, chronic injury
E. stress fractures, calcium deficiency, oligomenorrhea

68. A 30-year-old male presents with the complaint of back pain that has become worse over the past
3 months. He reports no recent injury or trauma. On examination while the patient is supine with his
right knee and hip flexed, the left knee comes off the table. The most likely diagnosis is

A. quadratus lumborum spasm left


B. piriformis spasm left
C. piriformis spasm right
D. psoas spasm left
E. psoas spasm right

29
2018 ACOFP IN-SERVICE EXAMINATION

69. A 40-year-old female presents to the office with the complaint of low back pain. On examination, she
has an antalgic gait and her torso is rotated right and flexed forward. Decreased hip extension on the
right and decreased internal rotation of the hip on the left are also noted. FABERE test is negative.
Which right lower extremity muscle is most likely in spasm?

A. adductor magnus
B. gluteus maximus
C. piriformis
D. psoas
E. rectus femoris

70. A high school football player is evaluated on the sideline for weakness in his right arm after tackling
another player and feeling a popping in his shoulder. He states that his arm dropped down to his side,
he was unable to move it until he lifted it with his left arm, and he felt a clunk in his right shoulder. He
currently has some numbness and residual weakness. A right-sided apprehensive test is positive. T1
and T2 somatic dysfunctions are present. The most likely diagnosis is

A. brachial plexopathy
B. fractured clavicle
C. fractured humerus
D. shoulder dislocation
E. torn rotator cuff

71. Examination of a patient reveals a bucket handle inhalation dysfunction of ribs 2 through 8. Which of the
following muscles is most likely in spasm?

A. anterior scalene
B. latissimus dorsi
C. pectoralis minor
D. posterior scalene
E. serratus anterior

30
2018 ACOFP IN-SERVICE EXAMINATION

72. A 44-year-old female presents to the office with vague vaginal symptoms. She is postmenopausal with
no apparent vaginal bleeding. There is no pain, but she states that she has mild abdominal distention
and urinary frequency. She reports intermittent episodes of diarrhea and constipation. A Pap smear is
negative for any intraepithelial lesions. The most likely diagnosis is

A. uterine fibroma
B. cervical carcinoma
C. endometriosis
D. ovarian malignancy
E. uterine sarcoma

73. A 40-year-old male presents with the abrupt onset of unilateral flank pain and nausea. Physical
examination reveals positive costovertebral angle tenderness. After obtaining a urinalysis, the most
definitive and cost effective test to evaluate this patient's condition is which of the following?

A. helical noncontrast CT scan


B. intravenous urography
C. MRI
D. plain film radiography of the abdomen
E. renal ultrasonography

74. A 34-year-old female presents with the complaint of pelvic pressure, heavy menstrual bleeding, and low
back pain. Her menses occur regularly every 30 days. Bimanual examination reveals an asymmetrical
uterus with a mass that is approximately 7 cm in diameter and moves with the cervix. Laboratory
studies reveal a negative serum HCG. Which of the following is correct regarding this patient's
condition?

A. cryomyolysis has been proven to be an effective treatment


B. masses of this type always arise within the serosal surface of the uterus
C. masses of this type have minimal malignant potential
D. oral contraceptives cannot be used in patients with this condition
E. pregnancy does not affect masses of this type

31
2018 ACOFP IN-SERVICE EXAMINATION

75. A 19-year-old female presents with the complaint of a vaginal discharge. History reveals that she is
sexually active. Physical examination reveals a profuse, milky, nonadherent discharge that
demonstrates a fishy odor after alkalization with a drop of KOH. The most appropriate treatment is

A. butoconazole 2% (Gynazole-1®) 5 g intravaginally for three days


B. clindamycin (Cleocin®) cream 2% one applicator (5 g) intravaginally at bedtime for three days
C. fluconazole (Fluconazole®) 150 mg orally in a single dose
D. metronidazole (Flagyl®) 2 g orally in a single dose
E. metronidazole (Flagyl®) 500 mg orally twice daily for seven days

76. A 78-year-old male nursing home resident is brought to the emergency department with a productive
cough, fever, and tachypnea. History reveals that he has received his annual flu shot and
pneumococcal immunization. Chest radiograph reveals consolidated pneumonia. The most likely
organism to cause this patient's bacterial pneumonia is

A. adenovirus
B. Haemophilus influenzae
C. Mycoplasma pneumoniae
D. Mycoplasma tuberculosis
E. Streptococcus pneumoniae

77. Following a structural examination, a patient is diagnosed with an inhalation dysfunction pump handle at
rib 4. Which of the following best describes this patient's findings?

A. anterior narrowing of the intercostal space below the dysfunctional rib


B. anterior rib moves caudad and is restricted on exhalation
C. latissimus dorsi is targeted when treated with muscle energy
D. rib is depressed anteriorly
E. rib is elevated anteriorly

32
2018 ACOFP IN-SERVICE EXAMINATION

78. A 24-year-old gravida 1 para 1 female presents for a 6-week postpartum examination. She reports
constant, dull, bilateral lower quadrant abdominal pain with varying intensity. The pain is not always
associated with the same movements during times of increased intensity. Pelvic examination reveals no
abnormalities. Examination of the sacrum reveals bilateral deep sulci, equivocal seated flexion test,
negative lumbar spring test, and a negative sphinx test. The most likely diagnosis is

A. bilateral sacral extension


B. bilateral sacral flexion
C. right-on-right sacral torsion
D. idiopathic scoliosis
E. left-on-left sacral torsion

79. A 56-year-old male presents to the emergency department with a 3-day history of nausea, vomiting, and
worsening epigastric pain. His triglyceride level noted in the electronic medical record 1 month ago was
1,482 mg/dL. Current laboratory studies reveal an elevated lipase and amylase. A sympathetic
viscerosomatic reflex is most likely to be located at

A. T1-T4
B. T5-T9
C. T10-T12
D. T12-L1
E. L2-L3

80. You are investigating a treatment modality for a patient. In your literature search, you search the
Cochrane database and discover the studies reporting the results of treatment consist of 3 separate
retrospective, double blind randomized control studies with consistent results. You determine the level
of evidence of the studies to be level

A. I
B. II
C. III
D. IV
E. V

33
2018 ACOFP IN-SERVICE EXAMINATION

81. A 38-year-old African-American male presents to the office with a four-month history of a progressively
worsening cough that occurs throughout the day and keeps him awake at night. History reveals that he
was diagnosed with HIV seven years ago. The most appropriate initial step in the workup of this patient
is

A. bronchoalveolar lavage
B. CD4 count
C. CT scan of the chest
D. pulmonary function testing
E. upper endoscopy

82. Multidrug-resistant tuberculosis refers to a disease that is resistant to

A. ethambutol (Myambutol®) and isoniazid (Nydrazid®)


B. isoniazid (Nydrazid®) and pyrazinamide (Tebrazid®)
C. isoniazid (Nydrazid®) and rifampin (Rifadin®)
D. pyrazinamide (Tebrazid®) and ethambutol (Myambutol®)
E. rifampin (Rifadin®) and pyrazinamide (Tebrazid®)

83. A 40-year-old male presents to the emergency department for evaluation of tachycardia, elevated blood
pressure, tremors, and hallucinations. He reports that he has not consumed any alcohol in three days
but has been drinking "for years." This patient's alcohol addiction and withdrawal symptoms are an
example of

A. alcohol abuse syndrome


B. episodic alcohol use
C. intermittent alcohol use
D. physical dependence
E. psychological dependence

34
2018 ACOFP IN-SERVICE EXAMINATION

84. Before graduating and accepting a job, you begin comparing relative value units for different areas of
the country. The three factors the resource-based relative value scale determines prices on are

A. demand of service, quality of service, and specialty level of care


B. effect of care, specialty level of care, and malpractice expense
C. professional service performed, practice expense, and malpractice expense
D. professional service performed, quality of service, and demand of service
E. practice expense, malpractice expense, and quality of service

85. In the geriatric patient, loss of balance and stability over time is most likely due to

A. cerebellar compromise from small strokes and transient ischemic attacks


B. decreased cognitive function
C. decreased function of visual, labyrinthine, and somatosensory centers
D. decreased hormone secretion from thyroid and gonads
E. loss of muscle mass with progressively unequal leg length

86. A 54-year-old male presents with the complaint of intermittent, painful, bloody diarrhea. A sacral
parasympathetic viscerosomatic reflex is most likely present if this patient has inflammatory pathology
of the

A. descending colon
B. duodenum
C. gallbladder
D. pancreas
E. terminal ileum

35
2018 ACOFP IN-SERVICE EXAMINATION

87. A male presents with a right testicular torsion. Palpable right-sided paravertebral viscerosomatic tissue
texture abnormalities and tenderness are most likely to be found at

A. T4-T5
B. T7-T8
C. T10-T11
D. L1-L2
E. L4-L5

88. A 27-year-old female presents with a history of worsening oligomenorrhea. She takes no routine
medications. She and her husband, while not attempting pregnancy, have never used contraceptives.
She is otherwise healthy and has no pertinent family history. Laboratory studies reveal:

Basic metabolic profile normal


Complete blood count normal
HCG negative

Following a thorough history and physical examination, the most appropriate initial study to confirm the
diagnosis is

A. cortisol challenge test


B. CT scan of the head with intravenous contrast
C. formal visual field testing
D. gadolinium enhanced MRI of the brain
E. random serum prolactin level

36
2018 ACOFP IN-SERVICE EXAMINATION

89. A 12-year-old male presents with a 2-day history of an abscess located on his right lower leg. On
examination he is afebrile and vital signs are normal. A circular fluctuant mass that measures 3 cm in
diameter is noted. An incision and drainage of the abscess is performed with no complications. The
most appropriate management is

A. intravenous vancomycin (Vanco®) 40 mg per kg divided into 4 doses for 10 days


B. no antibiotics
C. oral cephalexin (Keflex®) 250 mg twice daily for 10 days
D. oral clindamycin (Cleocin®) 10-20 mg per kg divided in 3 doses for 10 days
E. oral doxycycline (Doryx®) 100 mg twice daily for 10 days

90. The most common cause of occult bacteremia in children aged 3 months or older is

A. Chlamydia trachomatis
B. Haemophilus influenzae
C. Klebsiella pneumoniae
D. Neisseria meningitidis
E. Streptococcus pneumoniae

91. A 48-year-old male presents with the complaint of acute low back pain that radiates to the right foot. He
reports that he started tripping over his right foot today. Examination reveals weakness of knee
extension and decreased sensation at the medial aspect of the foot. The patellar reflex is diminished on
the involved side. The nerve root most likely affected is

A. L2
B. L3
C. L4
D. L5
E. S1

37
2018 ACOFP IN-SERVICE EXAMINATION

92. A 48-year-old female presents with the complaint of wrist pain. She reports that both of her wrists and
hands have hurt for the past six months. She states that the pain is worse early in the morning and then
she is able to "work through it" by midmorning. She denies any trauma. She reports no weight loss but
does admit to fatigue. Vital signs are normal. Physical examination reveals bilateral swelling of the
wrists and tenderness on palpation to her hands and fingers. Subcutaneous nodules are noted over the
extensor surfaces of her second and third fingers. Which of the following is most likely to confirm the
diagnosis?

A. anticyclic citrullinated peptide antibody


B. antinuclear antibody of 1:40
C. elevate leukocyte count on complete blood count
D. negative rheumatoid factor
E. osteophytes of the metacarpal joints on radiograph

93. Which of the following treatments has been shown to alter the long-term rate of decline in lung function
in patients with chronic obstructive pulmonary disease?

A. inhaled fluticasone (Flovent®)


B. inhaled ipratropium bromide (Atrovent®)
C. oral theophylline (Theo-Dur®)
D. oxygen therapy
E. smoking cessation

94. Which of the following is correct regarding the current recommendation for pneumococcal vaccination of
children?

A. pneumococcal conjugate vaccine (PCV13) should be administered to all children over five years of
age
B. pneumococcal conjugate vaccine (PCV13) should be administered to all children under two years of
age
C. pneumococcal conjugate vaccine (PCV13) should be withheld from children with asplenia
D. pneumococcal polysaccharide (PPV23) should be administered to all children over two years of age
E. pneumococcal polysaccharide (PPV23) should no longer be used for any children

38
2018 ACOFP IN-SERVICE EXAMINATION

95. A 78-year-old male presents to the office with a 24-hour history of acute lower abdominal pain. On
examination he appears uncomfortable and has an oral temperature of 38.0°C (100.4°F). Abdominal
palpation reveals involuntary guarding throughout his lower abdomen and discrete tenderness is elicited
over the tip of the 12th rib on the right. Which of the following is correct regarding this condition in elderly
patients?

A. CT scan is widely used as the imaging of choice


B. laparoscopic appendectomy is less beneficial than traditional open appendectomy
C. more likely to present for evaluation soon after the onset of symptoms
D. perforation and abscess formation are uncommon operative findings
E. typically exhibit symptoms of fever and right lower quadrant abdominal pain

96. A 21-year-old gravida 2 para 1 female at 23 weeks' gestation presents to the office for a prenatal
examination. History reveals an uneventful prior pregnancy and normal delivery two years ago. She has
no risk factors. Blood type is O+ and antibody screen is negative. Sexually transmitted disease testing
is negative. She reports that her grandmother has been diagnosed with type 2 diabetes mellitus and
asks if she should be tested. Screening for gestational diabetes in this patient should include a/an

A. 75 g oral glucose tolerance test between 24 and 28 weeks' gestation


B. fasting fingerstick and one-hour glucose tolerance test between 24 and 28 weeks' gestation
C. glycosylated hemoglobin level at this visit
D. random fingerstick blood glucose level test at this visit
E. three-hour glucose tolerance test between 24 and 28 weeks' gestation

97. A 26-year-old male presents to the office for evaluation of angry outbursts, mood swings, and vivid and
disturbing dreams that are interfering with his relationships. History reveals that he recently completed a
tour of duty in Afghanistan. The most appropriate treatment for this patient is

A. alprazolam (Xanax®)
B. aripiprazole (Abilify®)
C. duloxetine (Cymbalta®)
D. paroxetine (Paxil®)
E. risperidone (Risperdal®)

39
2018 ACOFP IN-SERVICE EXAMINATION

98. A 50-year-old female presents with the complaint of recurrent midback pain. Prior osteopathic
manipulative treatment has provided only limited success. She takes over-the-counter nonsteroidal
antiinflammatory medications daily for relief. Palpation and motion testing reveals rubbery paraspinal
tissue texture abnormalities and decreased rotation in the T7 region. These somatic dysfunctions are
resistant to high velocity, low amplitude treatment. The most likely underlying cause of this patient's
condition is

A. inhalation dysfunction of rib 7


B. peptic ulcer disease
C. T7 compression fracture
D. thoracic outlet syndrome
E. urolithiasis

99. A 71-year-old female presents to the office for an initial health maintenance examination. Her medical
records indicate that her last mammogram was two years ago. All of her previous mammograms have
been normal and she has no family history of breast cancer. Her last Pap smear was also two years
ago. She states that she has received a Pap smear every two to three years for most of her life. She
reports one abnormal Pap smear at age 55 that did not reveal cancer. Based on the current guidelines
for the U.S. Preventive Services Task Force (USPSTF), which of the following screening
recommendations would most likely apply to this patient?

A. mammogram and Pap smear screenings annually for life


B. mammogram screening for life and Pap smear screening is no longer recommended
C. mammogram screening is no longer recommended and Pap smear screening through age 74
D. mammogram screening through age 74 and Pap smear screening is no longer recommended
E. neither mammogram nor Pap smear screenings are recommended after age 70

40
2018 ACOFP IN-SERVICE EXAMINATION

100. A 35-year-old female presents to the office with a 10-day history of acute onset polydipsia. She has a
1-year history of type 2 diabetes mellitus and a 5-year history of autoimmune hypothyroidism. She
reports that since the onset of polydipsia, her fasting fingerstick blood glucose readings have been
elevated and range from 180-220 mg/dL. She also reports adherence to her prescribed medications,
diet, and exercise regimen. Her last glycosylated hemoglobin 2 months ago was 6.4%. You suspect
she may have an autoimmune component to her diabetes mellitus. Which of the following are
distinguishing characteristics of this patient's condition?

A. acute symptom of polyuria, intentional weight loss, and a personal history of autoimmune disease
B. unintentional weight loss, acute symptom of polydipsia, and a body mass index greater than
25 kg/m2
C. body mass index less than 25 kg/m2, acute symptom of polydipsia, and abnormal weight gain
D. family history of autoimmune disease, a body mass index greater than 25 kg/m2, and acute
symptom of polyuria
E. personal history of autoimmune disease, unintentional weight loss, and a body mass index less than
25 kg/m2

101. A 17-year-old female competitive gymnast presents to the office following the onset of low back pain.
History reveals no fractures. Her menses have been normal since menarche at age 13. On physical
examination the pain does not radiate and seems to be isolated to the center of the back. Range of
motion is greater than normal in all planes; however, extension is painful. Her strength is normal as
well as her reflexes. She has point tenderness on the spinous process of L4. Body mass index is
18 kg/m2. Radiograph of the lumbar spine reveals a Scotty dog sign on L4 on the left but is
unremarkable for any additional findings. A nuclear bone scan reveals increased activity in the area of
the defect. Which of the following is correct regarding the findings in this patient?

A. an unstable fracture that must be surgically repaired


B. common in athletes with hyperextension of the lumbar spine
C. consistent with the female athlete triad
D. no association with a familial pattern
E. unstable and would prohibit any further activity in gymnastics

41
2018 ACOFP IN-SERVICE EXAMINATION

102. An 8-year-old female is brought to the office with a 5-day history of wheezing and cough. Vital signs
reveal a temperature of 38.1°C (100.5°F) and a respiratory rate of 22/min. Physical examination is
unremarkable and oxygen saturation is 92% on room air. The wheezing is unresponsive to
bronchodilator treatment in the office. The most appropriate next step is

A. barium swallow
B. bronchoscopy
C. chest radiography
D. CT scan of the chest
E. pulmonary function studies

103. A 12-year-old male recently diagnosed with attention deficit hyperactivity disorder is prescribed
methylphenidate (Concerta®) 18 mg daily. He has a history of controlled seizures. Which of the
following should be obtained prior to initiating medication?

A. chemistry profile
B. complete blood count
C. ECG
D. thyroid-stimulating hormone level
E. urine drug screen

104. An otherwise healthy 86-year-old female presents to the office with painful lumbosacral somatic
dysfunction. Initial care considerations for this patient should include

A. obtaining a bone densitometry before using myofascial techniques


B. obtaining postural radiographs and making use of heel lifts
C. premedicating with an appropriate analgesic
D. scheduling longer intervals between treatments
E. scheduling longer osteopathic manipulative treatment sessions

42
2018 ACOFP IN-SERVICE EXAMINATION

105. A 41-year-old male presents after taking oral opiates for many years due to chronic back pain. He
wishes to discontinue these medications, but reports that every time he tries to stop he gets very ill and
shaky. He is concerned that he has a drug addiction. You advise him that according to the American
Psychiatric Association, he has an addiction if

A. he becomes shaky when discontinuing opioids


B. he receives a diminished effect from his current opioid dose
C. he requires both long-acting and short-acting opioids to control symptoms
D. he switches to methadone (Dolophine®) for pain control
E. his opioid use is causing recurrent social and interpersonal problems

106. A 48-year-old African-American male presents to the office for a preventive health examination after he
was discovered to have an elevated blood pressure at his place of employment. His log reveals
average daily readings of 144/90 mmHg over the past several weeks. His physical examination is
normal. Vital signs reveal a blood pressure of 142/90 mmHg. The most appropriate initial treatment for
this patient is

A. ACE inhibitors
B. β-blockers
C. α-blockers
D. combination therapy
E. thiazide diuretics

107. A 45-year-old male with type 2 diabetes presents for evaluation. He does not smoke. Current
medications include metformin (Glucophage®) 850 mg twice daily and atorvastatin (Lipitor®) 40 mg
daily. His blood pressure averages 134/84 mmHg. Laboratory studies reveal a low-density lipoprotein
level of 70 mg/dL, a glycosylated hemoglobin of 6.8%, and a urinary albumin/creatinine ratio of
80 μg/mg. The most appropriate recommendation for this patient is to

A. add amlodipine (Norvasc®)


B. add benazepril (Lotensin®)
C. continue his current regimen and recheck all laboratory studies in 3 months
D. increase metformin (Glucophage®) to 1,000 mg twice daily
E. repeat the urinary albumin/creatinine ratio in one year

43
2018 ACOFP IN-SERVICE EXAMINATION

108. Medicare offers coverage for which of the following routine services?

A. adult foster care placement


B. annual gynecologic examination
C. dental care
D. mammography
E. screening metabolic profile

109. A 31-year-old female presents to the office with a two-day history of a painless, red, right eye. She
denies any history of trauma. Her visual acuity is 20/25 in the right eye and 20/20 in the left eye.
Physical examination is significant for a large subconjunctival hemorrhage in the right eye. The most
appropriate next step is

A. diclofenac (Voltaren®) ophthalmic eye prescription


B. observation
C. urgent ophthalmologic referral
D. workup for coagulopathy
E. workup for secondary hypertension

110. A 33-year-old gravida 5 para 4 female presents at 39 weeks' gestation for induction of labor due to
preeclampsia superimposed on mild, chronic hypertension. Oxytocin (Pitocin®) is utilized for induction,
resulting in an uncomplicated labor and delivery of a 3.6-kg (8-lb) male. Delivery of the placenta occurs
40 minutes later. Despite vigorous uterine fundus massage, the patient has significant brisk bleeding.
The most likely cause of the postpartum hemorrhage is

A. atonic uterus
B. bleeding disorder
C. cervical laceration
D. retained placenta
E. uterine rupture

44
2018 ACOFP IN-SERVICE EXAMINATION

111. A 27-year-old trauma victim is transferred to the intensive care unit after sustaining multiple fractures in
a motorcycle accident. He is stabilized, and has received 2 units of packed red blood cells. Vital signs
reveal a blood pressure of 100/60 mmHg, a heart rate of 120/min, and a respiratory rate of 15/min.
Laboratory studies reveal a hemoglobin of 10.1 g/dL and a creatinine of 2.3 mg/dL. The remaining
laboratory studies are unremarkable. The most appropriate treatment for this patient's acute kidney
injury is to

A. administer dobutamine 2 mcg/min drip


B. administer fluid bolus with 1 L normal saline
C. administer intravenous furosemide (Lasix®) 40 mg followed by saline flush
D. initiate total parenteral nutrition
E. transfuse 1 additional unit of packed red blood cells

112. A 19-year-old female presents to the office with a 2-week history of vaginal discharge and pain on
urination. She admits to being sexually active with multiple male partners and that her partners did not
always use condoms. Which of the following would be the most likely infectious disease based on both
her symptoms and statistics on reportable sexually transmitted diseases in this patient's age group?

A. chlamydia
B. gonorrhea
C. herpes simplex
D. syphilis
E. trichomoniasis

45
2018 ACOFP IN-SERVICE EXAMINATION

113. An 87-year-old female presents to the office for evaluation of fatigue and pagophagia. Physical
examination is pertinent for skin and conjunctival pallor and angular cheilosis. Laboratory studies
reveal:

Hemoglobin 8.2 g/dL


Hematocrit 23.6%
Mean corpuscular hemoglobin 18 pg
Mean corpuscular volume 50 mcm3

The most likely findings for serum iron studies in this patient would be

A. decreased iron level, decreased total iron binding capacity, and decreased ferritin level
B. decreased iron level, decreased total iron binding capacity, and increased ferritin level
C. decreased iron level, increased total iron binding capacity, and decreased ferritin level
D. decreased iron level, increased total iron binding capacity, and increased ferritin level
E. increased iron level, increased total iron binding capacity, and decreased ferritin level

114. An 18-year-old female presents to the office to confirm a pregnancy. She is certain of the FDLMP,
which calculates to an intrauterine pregnancy of 6 weeks' gestation. Her prenatal care and counseling
for this visit should include

A. transabdominal ultrasound
B. glucose testing
C. intramuscular ceftriaxone (Rocephin®) 500 mg
D. Rh screening
E. tetanus, diphtheria, pertussis (Tdap) booster

46
2018 ACOFP IN-SERVICE EXAMINATION

115. A patient presents with a DXA scan T-score of -2.2. According to the World Health Organization, this
patient's score correlates with a diagnosis of

A. normal bone density


B. osteitis desiccans
C. osteogenesis imperfecta
D. osteopenia
E. osteoporosis

116. An 80-year-old male is brought to the office by his daughter for evaluation of depression. She states
that he is not tearful or sad, but seems disinterested and apathetic. She also notes that his short-term
memory is worsening, and he is having trouble concentrating. He states nothing is wrong with him and
that he is unsure why he is at this appointment today. He has no history of depression. Daily
medications include a multivitamin. His Mini-Mental State Examination score is 16/30 and his Geriatric
Depression Rating Scale score is 1/15. The most likely diagnosis is

A. Alzheimer disease
B. bipolar disorder
C. major depressive disorder
D. Parkinson syndrome
E. Pick disease

117. A 6-year-old male is brought to the office by his parents, who are concerned about his height. They
report that he is one of the shorter children in his class, but he is active and well. Birth history is
unremarkable. Physical examination reveals no significant abnormalities. A bone age is obtained and
the report documents that his bone age is 9 months less than his chronologic age (less than 2 standard
deviations for age mean). The most likely diagnosis is

A. congenital adrenal hyperplasia


B. constitutional growth delay
C. familial short stature
D. rickets
E. skeletal dysplasia

47
2018 ACOFP IN-SERVICE EXAMINATION

118. A 38-year-old male with a history of alcoholism presents with nausea, vomiting, and diffuse abdominal
pain that radiates to his back. Which of the following laboratory values is most suggestive of a poor
prognosis in this patient?

A. anion gap of 10 mEq/L


B. diastolic blood pressure greater than 90 mmHg
C. elevated serum amylase
D. elevated serum lipase
E. leukocyte count of 20.0 × 103/mcL

119. A 78-year-old male presents to the office with a 3-day history of gradually worsening nausea and
midabdominal pain. He denies rectal bleeding or weight loss. He has a temperature of 38.0°C
(100.4°F). Physical examination reveals mild constipation in association with left lower quadrant pain.
Laboratory studies reveal a leukocyte count of 11.2 × 103/mcL. The most likely diagnosis is

A. acute cholecystitis
B. acute diverticulitis
C. acute pancreatitis
D. small bowel obstruction
E. irritable bowel syndrome

120. A 49-year-old female with a past medical history significant for hypertension and obesity presents to
the office for follow up of her blood pressure. At home readings are normally around 150/95 mmHg and
it is approximately the same today. Current medications include metoprolol (Lopressor®), amlodipine
(Norvasc®), hydrochlorothiazide (HydroDIURIL®), and losartan (Cozaar®), all at the maximal doses.
She states that she is compliant and you believe her, and you have ruled out secondary hypertension.
The most appropriate treatment for this patient is

A. atenolol (Tenormin®)
B. prazosin (Minipress®)
C. spironolactone (Aldactone®)
D. terazosin (Hytrin®)
E. valsartan (Diovan®)

48
2018 ACOFP IN-SERVICE EXAMINATION

121. Which of the following classes of antihypertensive drugs is most likely to cause angioedema?

A. ACE inhibitors
B. α-blockers
C. β-blockers
D. calcium channel blockers
E. thiazide diuretics

122. A 56-year-old white female presents to the office for a health maintenance examination. She is
concerned because her younger sister recently suffered a myocardial infarction. She states that she
eats a healthy, low-fat diet, is a nonsmoker, and exercises most days of the week. Vital signs are
normal, with a blood pressure of 118/72 mmHg. Laboratory studies reveal:

Total cholesterol 216 mg/dL


HDL cholesterol 65 mg/dL
LDL cholesterol 151 mg/dL
Triglycerides 178 mg/dL

Her ASCVD risk is 1.7%. The most appropriate recommendation for cholesterol management is

A. high-dose statin therapy and therapeutic lifestyle changes


B. niacin (Niaspan®) and omega-3 fish oils
C. red yeast rice and therapeutic lifestyle changes
D. low-dose statin therapy and therapeutic lifestyle changes
E. continue therapeutic lifestyle changes only

49
2018 ACOFP IN-SERVICE EXAMINATION

123. A 58-year-old male with a history of several medical conditions is diagnosed with cellulitis of his lower
extremities. Due to multiple drug allergies you decide to treat the patient with ciprofloxacin (Cipro®).
After reviewing his medication list, which of the following medications should be discontinued or used in
an alternative dosing schedule to avoid decreased bioavailability of the fluoroquinolones?

A. digoxin (Lanoxin®)
B. hydrochlorothiazide (HydroDIURIL®)
C. paroxetine (Paxil®)
D. sucralfate (Carafate®)
E. verapamil (Calan®)

124. A 37-year-old female presents to the office with the acute onset of right eye pain. She describes the
pain as sharp and constant in nature that is worse when she turns her head to look to the right. The
most likely somatic dysfunction causing this patient's symptoms is

A. OA extended, sidebent right, rotated left


B. compression of the sphenobasilar synchondrosis
C. externally rotated right temporal bone
D. OA extended, sidebent left, rotated left
E. T2 extended, rotated right, sidebent right

125. A patient presents with chest pain associated with myocardial ischemia. The palpable anterior
Chapman reflex point in this patient is most likely to be found

A. in the center of the manubrium


B. in the intercostal space between ribs 2-3 adjacent to the sternum
C. in the intercostal space between ribs 4-5 at the sternocostal junction
D. in the intercostal space between ribs 2-3 in the midclavicular line
E. in the intercostal space between ribs 4-5 in the midclavicular line

50
2018 ACOFP IN-SERVICE EXAMINATION

126. A 10-year-old male is brought to the office by his mother for a health maintenance examination. On
questioning, his mother states that she is concerned that her son does not have many friends at school
and gets upset at home if a certain morning routine is not followed. She notes that he met all of his
developmental milestones appropriately, and that during a school screening his IQ was 110. The child
seems totally disinterested during the interview; however, he perks up when asked about what his likes
are and talks incessantly about dinosaurs during the rest of the examination. Physical examination is
unremarkable. The most likely diagnosis is

A. autism spectrum disorder level 1


B. autism spectrum disorder level 3
C. childhood disintegrative disorder
D. Rett syndrome
E. trisomy 18 syndrome

127. A 34-year-old female with a history of systemic lupus erythematosus presents to the office with the
complaint of acute pain in her left knee with noticeable swelling, warmth, and erythema. She denies
recent trauma or overuse. She is currently taking hydroxychloroquine (Plaquenil®) and prednisone
(Deltasone®). Her other lupus symptoms have been well controlled. The most likely cause of this
patient's knee pain is

A. infectious arthritis
B. patellofemoral syndrome
C. rheumatoid arthritis
D. Sjögren syndrome
E. systemic lupus erythematosus flare

51
2018 ACOFP IN-SERVICE EXAMINATION

128. A child is brought to the office by his parents for evaluation of a salmon pink rash and intermittent daily
temperatures spiking greater than 38.5°C (101.3°F) over the past 3 weeks along with the onset of pain
and inflammation of the joints. Physical examination reveals hepatosplenomegaly and
lymphadenopathy. Laboratory studies reveal a leukocyte count of 25 × 103/mcL, thrombocytosis,
profound anemia, and an elevated erythrocyte sedimentation rate and C-reactive protein level. The
most likely diagnosis is

A. Gaucher disease
B. Hunter syndrome
C. Hurler syndrome
D. Langerhans cell histiocytosis
E. systemic-onset juvenile idiopathic arthritis

129. The most appropriate pharmacologic treatment for children with enuresis is

A. atomoxetine (Strattera®)
B. bupropion (Wellbutrin®)
C. desmopressin (DDAVP®)
D. fluoxetine (Prozac®)
E. valproic acid (Depakote®)

52
2018 ACOFP IN-SERVICE EXAMINATION

130. A 62-year-old male presents to the office following the gradual onset of erectile dysfunction for the past
6 months. He is unable to achieve a satisfactory erection to have sexual intercourse with his wife. He
has noticed increased fatigue but attributes it to lack of physical activity during the winter season. Past
medical history reveals obstructive sleep apnea, benign prostatic hyperplasia, and a left breast
lumpectomy. He is not on any medications. He has occasional headaches that are relieved with
acetaminophen (Tylenol®). He denies alcohol consumption and does not smoke cigarettes. Physical
examination reveals decreased muscle mass and thinning hair. The remainder of the examination is
unremarkable. The most appropriate next step to rule out the primary cause of this patient's condition is
to

A. confirm empiric use and efficacy of phosphodiesterase-5 inhibitors


B. obtain a luteinizing hormone level
C. obtain a prolactin level
D. obtain an early morning testosterone level
E. order scrotal ultrasonography

131. A 45-year-old female with a history of generalized anxiety disorder presents with complaints of
insomnia, uncontrollable sweating, and increasing anxiety over the past 3 hours. History reveals
that 5 days ago, while on vacation, she ran out of her lorazepam (Ativan®) and has not refilled her
prescription. She is currently prescribed buspirone (BuSpar®) 30 mg twice daily and lorazepam
(Ativan®) 1 mg once to twice daily as needed and had been taking it regularly for approximately the
past 2 months. Vital signs reveal:

Temperature 37.1°C (98.8°F)


Blood pressure 138/84 mmHg
Heart rate 120/min
Respiratory rate 18/min

Examination reveals an anxious-appearing female with a sweaty forehead and tachycardia. The
remainder of the examination findings are unremarkable. The appropriate treatment for this patient is to

A. give flumazenil (Romazicon®)


B. give quetiapine fumarate (Seroquel®)
C. increase the buspirone (BuSpar®)
D. restart lorazepam (Ativan®)
E. send her to the hospital

53
2018 ACOFP IN-SERVICE EXAMINATION

132. A 35-year-old male presents to the office with the complaint of shoulder pain for several weeks. The
pain began shortly after he fell on the shoulder during a rugby match and is localized to the anterior
shoulder and deltoid areas. The pain is aggravated by reaching behind him. Examination reveals no
deformity, but he has pain when the shoulder is adducted to 90 degrees and the flexed elbow is
brought across the body to the midline. There is no pain with abduction. Radiograph reveals slight
superior displacement of the clavicle relative to the acromion. The most appropriate initial management
of this patient's injury is

A. a sling for comfort, ice, and antiinflammatory medications for 3 to 6 weeks


B. physical therapy to improve range of motion and reduce long-term complications
C. referral to an orthopedist to expedite operative repair
D. a clavicle strap (figure 8) for 3 to 6 weeks
E. an ultrasound to rule out occult fracture or ligament damage

133. A 28-year-old gravida 1 para 0 female presents to the clinic with the complaint of vaginal bleeding.
Following an obstetrical ultrasound, the radiologist calls to report that she has a threatened abortion.
Current evidence-based medicine states which of the following?

A. evidence confirms that bedrest is the treatment of choice


B. limited physical activity is advised
C. obtain maternal blood type
D. obtain serial HCGs
E. treatment should include progesterone or synthetic progestins

134. Which of the following components is necessary for an accurate diagnosis of attention deficit
hyperactivity disorder in a child?

A. abnormal parental Conners Rating scores


B. academic underachievement
C. symptoms present in more than one setting
D. lack of hostility
E. normal family relationships

54
2018 ACOFP IN-SERVICE EXAMINATION

135. A 6-year-old male is brought to the office by his mother for assessment of behavior problems. History
reveals that he is failing first grade and has had behavior issues since infancy. Neither of his parents
completed high school and his father had a diagnosis of attention deficit hyperactivity disorder as a
child. His parents report that he is disruptive, not respectful of adults, actively defies their requests, and
argues with adults that are not custodial. He also deliberately annoys others and is vindictive. The most
likely diagnosis is

A. antisocial personality disorder


B. attention deficit disorder
C. conduct disorder
D. oppositional defiant disorder
E. pervasive developmental disorder

136. A 32-year-old female with a history of gestational diabetes has a prolonged second stage of labor. After
delivery of the fetal head, the head retracts onto the perineum. Which of the following is the most
appropriate management at this time?

A. attempt to rotate the fetal shoulder posteriorly


B. forceps-assisted delivery
C. fundal pressure
D. general anesthesia for the mother
E. McRoberts maneuver

137. A 39-year-old gravida 2 para 2 female presents to the office with the complaint of irregular menses for
3 to 4 months. Prior periods were regular but now for a few days she will spot, then nothing, and then
she will have a heavy flow for a few days. She can even go a few weeks without any bleeding. She has
a body mass index of 28 kg/m2, but she reports no recent weight gain. Home pregnancy testing has
been negative, as reported by her. History and examination are otherwise unremarkable. She is most
likely experiencing which type of bleeding?

A. anovulatory bleeding
B. bleeding from a coagulation disorder
C. bleeding from anatomical abnormalities
D. bleeding from threatened abortion
E. infectious bleeding

55
2018 ACOFP IN-SERVICE EXAMINATION

138. A 9-year-old male presents to an urgent care center for evaluation of severe sore throat, malaise, and
low-grade fever. He is given amoxicillin (Amoxil®). He presents 3 days later with the same symptoms.
He is still febrile, his throat pain is worse, and he is extremely congested. Physical examination reveals
a maculopapular rash on the trunk, arms, and legs. Laboratory studies are most likely to reveal

A. elevated BUN
B. elevated IgE levels
C. low serum albumin
D. lymphocytosis with atypical lymphocytes
E. reduced complement CH50

139. A 14-year-old female is brought to the office with the acute onset of right facial weakness and eyelid
droop that began 1 week after returning from summer camp. She has no pain and is unaware of any
environmental exposure. She has no significant medical history. Physical examination reveals no
abnormalities except for her facial findings. The most likely diagnosis is

A. cryptococcus infection
B. Giardia exposure
C. Guillain-Barré syndrome
D. Rocky Mountain spotted fever
E. Lyme disease

140. An 85-year-old female presents to the office with increased urinary loss. She also noted feeling a bulge
in her vagina. She has previously tried pelvic floor exercises and pelvic floor physical therapy, which
helped her incontinence. Past medical history reveals oxygen-dependent chronic obstructive
pulmonary disease and heart failure. Pelvic examination reveals a normal size uterus that prolapses
with a Valsalva maneuver. The most appropriate next step is to

A. fit her for a pessary


B. initiate mirabegron (Myrbetriq®)
C. initiate oxybutynin (Ditropan®)
D. perform a sling procedure
E. perform a urethropexy

56
2018 ACOFP IN-SERVICE EXAMINATION

141. A 65-year-old male presents to the office with the complaint of a worsening cough after being placed
on oral corticosteroids by an urgent care physician. He also complains of purulent sputum over the past
7 days. He was recently diagnosed with chronic obstructive pulmonary disease (GOLD 1) and has
been using a short-acting bronchodilator regularly. The most appropriate next step in the management
of this patient's condition is to

A. add a long-acting bronchodilator to his regimen


B. add an inhaled corticosteroid to his regimen
C. recommend home oxygen therapy
D. admit for hospitalization since he has failed outpatient therapy
E. prescribe oral macrolide antibiotics

142. A 37-year-old female presents to the clinic for evaluation of dysuria, urinary frequency, and mild flank
pain. She reports that her symptoms have gradually worsened over the past few weeks. Past medical
history reveals urinary tract infections, but she reports that she has not been to a clinic for years. Vital
signs are normal. Physical examination reveals flank pain on the right side. Urinalysis reveals
erythrocytes, positive nitrites, and a pH of 7.2. She is prescribed empirical antibiotics and asked to
follow up in 2 weeks. Cultures come back showing Proteus mirabilis. On follow-up evaluation, the
patient reports that her symptoms have barely improved and she is still experiencing flank pain. A
CT scan without contrast confirms the diagnosis. The most appropriate management for this patient is

A. low-sodium diet
B. increased fluid intake and analgesics
C. allopurinol (Zyloprim®)
D. hydrochlorothiazide (Microzide®)
E. surgical intervention

57
2018 ACOFP IN-SERVICE EXAMINATION

143. A 45-year-old male presents to the outpatient clinic with a 2-day history of right foot pain. The
symptoms began after playing cards and drinking 12 beers. Current medications include
hydrochlorothiazide (HydroDIURIL®) 25 mg daily to control his blood pressure. Past medical history is
otherwise unremarkable. On physical examination the right metatarsophalangeal joint is red, swollen,
erythematous, warm to palpation, and very tender. The most appropriate treatment for this patient is

A. acetaminophen (Tylenol®) 650 mg every 6 hours for 5 days


B. allopurinol (Zyloprim®) 300 mg daily for 10 days
C. cephalexin (Keflex®) 500 mg every 6 hours for 10 days
D. hydrocodone/APAP (Norco®) 5/300 mg every 6 hours for 5 days
E. naproxen (Naprosyn®) 500 mg twice daily for 10 days

144. Medroxyprogesterone acetate (Depo-Provera®) has been shown to correlate to increased risk for

A. anxiety
B. electrolyte imbalance
C. endocervical cancer
D. endometriosis
E. osteoporosis

145. A 12-year-old female presents to the office with a puncture wound to the hand. History reveals that she
was bitten while playing with her cat. After cleaning and examining the wound, you note no additional
injury to the hand. The most appropriate antibiotic prophylaxis for this patient is

A. amoxicillin/clavulanate (Augmentin®)
B. azithromycin (Zithromax®)
C. cephalexin (Keflex®)
D. clindamycin (Cleocin®)
E. nitrofurantoin (Macrobid®)

58
2018 ACOFP IN-SERVICE EXAMINATION

146. A 7-year-old previously well male is brought to the office for evaluation of a skin rash that developed
this morning over his buttocks and legs in a "waist down" distribution. The rash began with petechiae
that soon coalesced into purplish areas. There is some pruritus with the rash. His face and hands are
spared. He complains of colicky abdominal pain but no nausea or vomiting. He has a low-grade fever
today with malaise and headache. His mother reports a "cold" 2 weeks ago that resolved
spontaneously. Physical examination reveals mild swelling of the ankle joints and knees, although no
warmth or erythema is noted. The most likely diagnosis is

A. Henoch-Schönlein purpura
B. molluscum contagiosum
C. Kawasaki syndrome
D. rheumatic fever
E. systemic lupus erythematosus

147. A 5-month-old male is brought to the office for evaluation of a cough. His mother reports that for the
past week he has been coughing in bursts. He tries to catch his breath after coughing with deep,
whooping inhalations. He often vomits after coughing. The most appropriate first-line medication class
to prescribe is

A. antivirals
B. broad-spectrum cephalosporin
C. fluoroquinolones
D. immunoglobulins
E. macrolides

148. A healthy 12-year-old female is evaluated for a preparticipation sports physical examination. She has
previously received all recommended vaccinations through age 5. The most appropriate vaccination for
this patient is

A. diphtheria and tetanus (DT) and FluMist® (influenza)


B. Gardasil® (HPV) only
C. tetanus, diphtheria, pertussis (Tdap) only
D. tetanus, diphtheria, pertussis (Tdap), meningococcal conjugate (MCV4), and Gardasil® (HPV)
E. varicella (Varivax®), measles, mumps, rubella (MMR), and diphtheria and tetanus (DT)

59
2018 ACOFP IN-SERVICE EXAMINATION

149. A 29-year-old male presents for a routine pre-employment physical examination. He admits to drinking
2 to 3 beers every night and a 12 pack on weekends. He has never had a DUI, and does not feel that
his drinking impedes his work or relationships. He has admitted to thinking about cutting down on his
drinking, and that his wife would like him to drink less. History and physical examination are otherwise
unremarkable. The most appropriate treatment for this patient is to

A. empathetically counsel him regarding safe drinking guidelines and provide counseling resources
B. hospitalize immediately for alcohol detoxification and inpatient treatment
C. recommend immediate alcohol cessation, prescribe chlordiazepoxide (Librium®), and schedule a
follow-up appointment in 1 week
D. recommend immediate alcohol cessation, prescribe disulfiram (Antabuse®), and schedule a follow-
up appointment in 1 week
E. screen for psychiatric disorders and inform him that additional counseling or treatment is
unnecessary

60
2018 ACOFP IN-SERVICE EXAMINATION

150. A 74-year-old female with a history of rheumatoid arthritis presents to the office to establish care. She
is currently using nonsteroidal antiinflammatory drugs intermittently for pain control. She takes a
calcium +D tablet twice daily. She is on no prescription medications. Vital signs reveal:

Temperature 38.0°C (100.4°F)


Blood pressure 128/78 mmHg
Heart rate 72/min
Respiratory rate 16/min

On physical examination heart is regular without murmur. Lungs are clear to auscultation bilaterally.
Musculoskeletal examination reveals swollen, boggy, proximal interphalangeal and
metacarpophalangeal joints bilaterally that are slightly tender to touch. Laboratory studies reveal:

Hemoglobin 11 g/dL
Hematocrit 33%
Mean corpuscular volume 84 mcm3
Red blood cell distribution width 41%
Ferritin 260 ng/mL
Iron 40 mcg/dL
Total iron-binding capacity 150 mcg/dL
Erythrocyte sedimentation rate 45 mm/h
Vitamin B12 425 pg/mL

The most appropriate initial step in this patient's management is

A. oral celecoxib (Celebrex®) 200 mg daily


B. intramuscular cyanocobalamin 1,000 mcg monthly
C. subcutaneous erythropoietin 100 units/kg 3 times weekly
D. oral hydroxychloroquine (Plaquenil®) 400 mg daily
E. intravenous iron sucrose (Venofer®) 200 mg every 3 weeks

61
2018 ACOFP IN-SERVICE EXAMINATION

151. A 62-year-old female presents to the office for a follow-up evaluation following a recent visit for fatigue.
She has a history of type 2 diabetes mellitus. Current medications include metformin 1,000 mg twice
daily and aspirin 81 mg daily. She takes no over-the-counter supplements. Physical examination
findings are normal. Laboratory studies reveal:

Hemoglobin 11.5 g/dL


Hematocrit 34%
Mean corpuscular volume 108 mcm3
Red blood cell distribution width 12%

Follow-up testing reveals macroovalocytes and hypersegmented neutrophils on peripheral smear. The
reticulocyte count is 1% and the vitamin B12 level is 500 pg/mL. The most appropriate next step in this
patient's work-up is

A. bone marrow biopsy


B. folic acid level
C. hepatic function panel
D. methylmalonic acid
E. thyroid-stimulating hormone level

152. A 79-year-old black male presents to the office for a follow-up evaluation. He was recently discharged
from the local hospital following an episode of transient ischemic attack. He has a history of
hypertension, diabetes, depression, and morbid obesity. He is currently taking metoprolol succinate
(Toprol XL®) 50 mg daily, metformin (Glucophage®) 1,000 mg twice daily, paroxetine (Paxil®) 20 mg
daily, aspirin (Bayer®) 81 mg daily, and a multivitamin. Which of the following medications is first-line
for secondary prevention in this patient?

A. clopidogrel (Plavix®) 75 mg
B. aspirin (Bayer®) 325 mg
C. amlodipine (Norvasc®) 10 mg
D. atorvastatin (Lipitor®) 80 mg
E. warfarin (Coumadin®) 5 mg

62
2018 ACOFP IN-SERVICE EXAMINATION

153. A 16-year-old male is brought to the office after experiencing a witnessed generalized seizure at home.
History reveals that he stayed up all night playing video games. He reports that he often "spaces out"
at school lately and is experiencing jerking in his arms when he first wakes up. His uncle has epilepsy.
The most likely diagnosis is

A. juvenile myoclonic epilepsy


B. absence seizures
C. benign rolandic epilepsy
D. benign occipital epilepsy
E. febrile seizures

154. An 88-year-old female is brought to the office by her son due to a 6-month history of a decreased
appetite. Her son reports that she once enjoyed going to the quilting club at the local senior center, but
she now prefers to stay at home and does not seem to get enjoyment from activities. He reports that
she skips meals and does not have an appetite. The patient reports that she has been having trouble
sleeping but otherwise does not have any concerns. She lives alone in her own home. She does
require some assistance from her son for some of her instrumental activities of daily living, including
driving and managing her finances. She cooks her own meals and manages her own medications. Past
medical history includes mild cognitive impairment and osteoarthritis. Current medications include a
daily multivitamin and acetaminophen 500 mg every 8 hours as needed for pain. Physical examination
reveals a frail female who weighs 50 kg (110 lb). She has lost 2.3 kg (5.0 lb) since her last visit
3 months ago. Her heart and lungs are normal to auscultation. Neurologic examination findings are
normal. The patient has a Patient Health Questionnaire-9 (PHQ-9) score of 9/27. The most appropriate
management for this patient is

A. cyproheptadine (Periactin®)
B. dronabinol (Marinol®)
C. mirtazapine (Remeron®)
D. increase size of food portions at meals
E. megestrol (Megace®)

63
2018 ACOFP IN-SERVICE EXAMINATION

155. A 25-year-old female admitted to the hospital for a cesarean section develops abdominal cramps and
nonbloody diarrhea following prophylactic intravenous antibiotic treatment. Laboratory studies are
positive for Clostridium difficile, and she is treated with oral metronidazole (Flagyl®) 500 mg 3 times
daily. Her diarrhea resolves satisfactorily and she is discharged home. Three days later she presents to
the office with complaints that her abdominal discomfort and diarrhea have returned. She reports that
she is able to tolerate oral intake and is comfortable. On physical examination she is afebrile, and has
no rebound tenderness or abdominal rigidity. The most appropriate next step is to

A. obtain laboratory and stool studies prior to initiating treatment


B. start vancomycin (Vancocin®) 750 mg intravenously
C. start loperamide (Imodium®) orally 2 mg
D. repeat metronidazole (Flagyl®) 500 mg orally 3 times daily
E. start ciprofloxacin (Cipro®) orally 250 mg twice daily

156. A 14-year-old female with a body mass index of 24 kg/m2 presents to the office for evaluation of painful
menses. She reports experiencing intermittent menstrual cramps since menarche at age 12, but that
they seem to be getting worse. They begin the day before her period starts, and usually last 2 days. In
recent months she has missed school because of her discomfort and associated diarrhea. She takes
ibuprofen (Advil®) 200 mg when the pain gets bad. She denies any history of sexual activity, and has
never had a pelvic examination. Past medical history does not include any associated or chronic
conditions. The most appropriate treatment is to

A. discontinue the ibuprofen (Advil®), perform a pelvic examination, and refer her to a gynecologist
B. increase the ibuprofen (Advil®) to 400 to 600 mg every 6 hours, starting the day before her period
and continuing for 2 days
C. discontinue the ibuprofen (Advil®), perform a pelvic examination, and consider placing an
intrauterine device
D. alternate ibuprofen (Advil®) with acetaminophen (Tylenol®) every 4 to 6 hours, starting the day
before her period and continuing for 2 days
E. add an antidiarrheal medication to the ibuprofen (Advil®) 200 mg dose, continuing for 2 days

64
2018 ACOFP IN-SERVICE EXAMINATION

157. A 24-year-old gravida 1 para 0 female at 41 weeks' gestation presents for induction because she is
post dates. Her pregnancy was complicated by first trimester bleeding that resolved without
intervention. Her cervix today is 4/50/-2. She would like to ambulate. Her initial electronic fetal
monitoring strip shows a category I tracing. The best advice to this patient is that she

A. will require continuous monitoring throughout her entire labor


B. can have intermittent monitoring throughout her entire labor
C. can have intermittent monitoring during stage 1 of labor
D. can have intermittent monitoring during stage 2 of labor
E. will require continuous monitoring during stage 1 of labor

158. A 66-year-old para 4 gravida 4 female presents with a 6-month history of vaginal itching and urinary
frequency. She reports no bleeding or discharge. She has been postmenopausal for 14 years. She
denies smoking. On examination, the vagina is pale but no lesions are seen. The introitus is normal.
Urinalysis reveals few erythrocytes and few leukocytes. The most appropriate treatment is vaginal

A. estrogen (Premarin®, estradiol cream)


B. cortisone
C. metronidazole (Flagyl®)
D. lactobacillus
E. petroleum jelly (Vaseline®) barrier

65
2018 ACOFP IN-SERVICE EXAMINATION

159.A 14-month-old female is brought to the clinic by her mother for evaluation of fussiness and decreased
appetite. She was previously healthy and takes no prescription medications. Her mother reports that
she normally attends daycare but has been home the last 2 days with a temperature of 38.3°C
(101.0°F). Physical examination reveals the rash shown in the exhibit. The rash began this morning
and covers her trunk and lower extremities. She also has several small, 1- to 2-mm lesions on her
mucous membranes that are slightly hemorrhagic. She appears uncomfortable when bending her legs
at the knees. The most appropriate next step is to

A. order blood cultures, cerebrospinal fluid studies and culture and empiric parenteral antibiotics for
meningeal pathogens
B. reassure the caregiver that this is a viral exanthem and will resolve spontaneously
C. monitor for fluid intake and, if unable to take fluids, may require hospitalization for intravenous
hydration
D. administer a single whole body treatment with permethrin cream (Elimite®)
E. administer intravenous gamma globulin and aspirin and obtain an echocardiogram

66
2018 ACOFP IN-SERVICE EXAMINATION

160. A 56-year-old female is brought to the emergency department by her husband with the gradual onset of
lethargy and confusion over the past few days. Initial laboratory studies reveal a normal complete blood
count, a normal comprehensive metabolic profile, and a calcium level of 14.5 mg/dL. Which of the
following is the most appropriate initial management?

A. volume expansion with saline and administration of salmon calcitonin and zoledronic acid
B. subcutaneous synthetic parathyroid hormone and vitamin D in doses of 50,000 IU
C. glucose in combination with insulin until serum calcium is below 10 mg/dL
D. surgical removal of the parathyroid glands
E. subcutaneous denosumab (Prolia®) at 60 mg after admission to the intensive care unit

161. A 3-month-old infant is brought to the emergency department with a 3-day history of nasal congestion,
fever, and a worsening cough. Vital signs reveal a temperature of 38.3°C (101.0°F), a respiratory rate
of 64/min, and an oxygen saturation of 89% on room air. A respiratory syncytial virus nasal wash is
noted to be positive. The most appropriate in-patient management includes intravenous fluids and

A. aerosolized bronchodilators
B. supplemental O2
C. azithromycin (Zithromax®)
D. intravenous ceftriaxone (Rocephin®)
E. intravenous steroids

162. A 10-year-old male presents with increasing shortness of breath and cough with exercise. He has a
history of asthma and uses his albuterol inhaler once or twice daily. The best objective measure of
daily control of this patient's asthma is

A. blood pressure
B. diary recordings of albuterol use
C. daily recordings of peak expiratory flow rate
D. pulse oximetry
E. respiratory rate

67
2018 ACOFP IN-SERVICE EXAMINATION

163. A 30-year-old female presents with a 2-day history of hematuria, dysuria, increased urinary frequency,
and nocturia. She has no fever, chills, or back pain. On examination she does not appear ill and there
is no costovertebral angle tenderness. She has a temperature of 37.5°C (99.5°F). The most effective
treatment is a

A. single dose of ampicillin (Omnipen®) and probenecid (Benemid®)


B. one-day course of trimethoprim-sulfamethoxazole (Bactrim®)
C. three-day course of trimethoprim-sulfamethoxazole (Bactrim®)
D. seven-day course of ampicillin (Omnipen®) and probenecid (Benemid®)
E. ten-day course of ampicillin (Omnipen®) and probenecid (Benemid®)

164. An 18-year-old male presents to the office with the concern that since he began college this year he
has not been performing to the level that he did in high school. He states that he is not completing his
exams because he is checking his answers on the answer sheet five to six times before moving on to
the next question. His mother has a history of anxiety disorder. The most likely diagnosis is

A. adjustment disorder
B. attention deficit disorder
C. cognitive impairment
D. dysthymia
E. obsessive-compulsive disorder

165. A 45-year-old male presents for evaluation of fever and rash. He reports a five-day history of fever,
headache, malaise, and nausea. Today he noticed a rash that affects his extremities and torso,
including his palms. The rash does not itch. History reveals that he recently returned from a fishing trip
in Georgia. The most likely diagnosis is

A. Hansen disease
B. Hantavirus
C. Lyme disease
D. Rocky Mountain spotted fever
E. tularemia

68
2018 ACOFP IN-SERVICE EXAMINATION

166. A 26-year-old female presents with a one-month history of tremor. She is three months post
uncomplicated vaginal delivery. Her only medication is norgestimate/ethinyl estradiol. The remainder of
her history is normal. Physical examination reveals a mild intention tremor of her hands bilaterally. Her
thyroid is neither enlarged nor tender, and the remainder of the ENT, neurologic, and mental status
examinations are normal. Laboratory studies reveal:

Complete blood count normal


Blood glucose normal
Calcium normal
Electrolytes normal
Thyroid-stimulating hormone 0.25 mcU/mL
Thyroxine 18 mcg/dL
T3 total serum 300 ng/dL
Radioactive iodine uptake low

The most likely diagnosis is

A. euthyroid sick syndrome


B. Graves disease
C. postpartum thyroiditis
D. secondary hypothyroidism
E. subclinical hyperthyroidism

167. A 43-year old female presents to the office for a health maintenance examination. Though
asymptomatic, she is found on examination to have a left-sided thyroid nodule. Laboratory studies
reveal a thyroid-stimulating hormone level of 7.5 mcU/ml and a normal free thyroxine level. An
ultrasound reveals a 1.5-cm solitary nodule of the left lobe. Which of the following is the most
appropriate next step?

A. fine-needle aspiration biopsy of the nodule


B. therapy with levothyroxine (Levoxyl®)
C. observation only with a repeat sonogram in 6 months
D. thyroid scintigram
E. surgical removal of the nodule

69
2018 ACOFP IN-SERVICE EXAMINATION

168. A 2-year-old female is brought to the office for evaluation of a rash. Her parents report that the child
had a low-grade fever, sore throat, and malaise 5 days ago that resolved after 48 hours. Today she
developed very rosy cheeks without return of fever or other symptoms. Physical examination confirms
the presence of a bilateral, erythematous facial rash. The remainder of the examination findings are
normal. The most likely cause of the rash is

A. erythema infectiosum
B. impetigo
C. pityriasis rosea
D. roseola
E. scarlet fever

169. A 32-year-old male presents to the office with the complaint of left heel itchiness. He notes large fluid-
filled lesions on the foot that have been present for the past few days. The lesions are not painful. He
denies fever, new socks, or exposure to allergens. Physical examination reveals dry and fissured
heels. The most likely diagnosis is

A. Coxsackievirus
B. contact dermatitis
C. dyshidrotic eczema
D. molluscum contagiosum
E. shingles

170. A 25-year-old gravida 2 para 2 female with a history of severe asthma is evaluated for postpartum
hemorrhage. Uterine atony is diagnosed and bimanual compression of the uterus is performed. Which
of the following could cause asthmatic exacerbation in this patient?

A. misoprostol (Cytotec®)
B. carboprost (Hemabate®)
C. methylergonovine (Methergine®)
D. ergonovine (Ergotrate®)
E. oxytocin (Pitocin®)

70
2018 ACOFP IN-SERVICE EXAMINATION

171. A 15-month-old male is brought to the office for evaluation of a low-grade fever, cough, and congestion
over the past 2 days. His parents describe the cough as "seal-like". The child is still eating and drinking
normally, and other than the coughing and mild fussiness, is acting like himself. His vital signs are
normal, with an oxygen saturation of 98% on room air. Lung examination reveals mild stridor but no
retractions or nasal flaring. What is the best treatment plan for this patient?

A. administer dexamethasone (Decadron®) 0.6 mg/kg once orally and educate the parents on when to
seek further medical attention
B. admit to the hospital for close observation
C. administer racemic epinephrine (Racepinephrine®) in the office
D. administer immediately 2 L/min of oxygen via nasal cannula
E. prescribe a methylprednisolone acetate (Medrol®) dose pack for outpatient therapy

172. During the initial months of residency training, first-year residents (regardless of specialty) have shown
a marked increase in symptoms for which disorder?

A. obsessive-compulsive disorder
B. depression
C. generalized anxiety disorder
D. narcolepsy
E. attention deficit disorder

173. A 37-year-old female presents for follow-up of hypertension that is poorly controlled on a thiazide
diuretic and β-blocker. She reports good compliance with her medication regimen and lifestyle
changes. Her home blood pressure readings confirm inadequate blood pressure control. Renal function
is normal. She is started on an angiotensin-converting enzyme inhibitor in addition to her other
medications. Repeat renal function testing 3 weeks later demonstrates a doubling of her serum
creatinine. This change in her creatinine is most likely due to the presence of what secondary cause for
her hypertension?

A. hyperthyroidism
B. hypothyroidism
C. renal artery stenosis
D. obstructive sleep apnea
E. polycystic kidney disease

71
2018 ACOFP IN-SERVICE EXAMINATION

174. A 3-year-old male with known sickle cell anemia presents to the emergency department with his
mother, who reports that he has been lethargic for several days. Physical examination is unremarkable,
with the exception of pale conjunctivae. Laboratory studies reveal aplastic anemia. Exposure to which
of the following is most likely to result in an aplastic crisis in this child?

A. coxsackievirus A16
B. human herpesvirus 6
C. parvovirus B19
D. rubeola
E. varicella zoster

175. A 37-year-old male presents to the office to establish care. History reveals that he has not seen a
physician since high school. Past medical history reveals that his mother was diagnosed with colon
cancer at the age of 45. He has no complaints and his review of systems is negative. Based on the
American Cancer Society recommendations, what information should this patient be given regarding
colon cancer screening?

A. a normal digital rectal examination with a negative fecal occult blood test eliminates the need for
screening
B. begin screening at the age of 40
C. begin screening when symptoms develop
D. no screening is necessary at his current age
E. obtain a colonoscopy now

72
2018 ACOFP IN-SERVICE EXAMINATION

176. A 66-year-old male presents to the office for a health maintenance examination. He has not had a
physical examination in 3 years. He is generally healthy and is up to date on his immunizations. Past
medical history reveals hypertension, for which he has taken hydrochlorothiazide for the past 5 years
with good control of his blood pressure. He started smoking between 1/2 to 1 pack per day in college
but quit approximately 10 years later. He denies cardiovascular, respiratory, gastrointestinal, and
genitourinary symptoms. Based on U.S. Preventive Services Task Force (USPSTF) recommendations,
which of the following should be offered to this patient?

A. low-dose CT scan of the lungs


B. ultrasound of the abdomen
C. ECG
D. carotid artery duplex study
E. prostate-specific antigen screening

177. A 69-year-old female presents for a routine health maintenance examination. Her last mammography
was 3 years ago with normal results and her last colonoscopy, which was negative for any findings,
was 6 years ago. She gets a flu shot annually but has never had a pneumonia vaccine. She had the
shingles infection last year but has not had a shingles vaccine. She had a tetanus, diphtheria, pertussis
(Tdap) vaccine 4 years ago. She underwent a total hysterectomy and bilateral salpingo-oophorectomy
at age 52 for dysfunctional uterine bleeding, which identified no cancer. She denies any complaints. A
recent community blood screen revealed that complete metabolic profile, lipids, and blood count were
all within the normal range. Based on current screening and preventive health guidelines, what is the
most appropriate recommendation for this patient?

A. bone densitometry screening, mammography, pneumococcal conjugate vaccine (PCV13), and


zoster vaccine
B. bone densitometry screening, colonoscopy, mammography, Pap smear, and pneumococcal
polysaccharide vaccine (PPSV23)
C. clinical breast examination, colonoscopy, pneumococcal polysaccharide vaccine (PPSV23), and
zoster vaccine
D. fecal occult blood testing, mammography, and tetanus and diphtheria (Td) vaccine
E. mammography; Pap smear; tetanus, diphtheria, pertussis (Tdap) vaccine; and zoster vaccine

73
2018 ACOFP IN-SERVICE EXAMINATION

178. A 10-year-old male is brought to the office by his mother with a 4-week history of a worsening rash that
started as a red, itchy area on his upper arm. His mother became concerned as it became larger and
used some over-the-counter creams on it, but the rash worsened. Further questioning reveals that the
creams used 1 week ago contained cortisone 1%. The child lives on a cattle farm and gets rashes
frequently. He recently had a head cold, but is otherwise healthy. Family history is negative for skin
disorders. Examination reveals a 4-cm erythematous, scaly patch with an annular well-defined border.
The most likely diagnosis is

A. contact dermatitis
B. erythema multiforme
C. annular psoriasis
D. tinea corporis
E. granuloma annulare

179. What is the recommendation for a dilated comprehensive eye examination performed by an
ophthalmologist or optometrist in a patient with type 1 diabetes?

A. within 5 years of diagnosis


B. within 7 years of diagnosis
C. after hemoglobin A1c is under 7%
D. after hemoglobin A1c is under 6%
E. at the time of diagnosis

74
2018 ACOFP IN-SERVICE EXAMINATION

180. A 5-year-old female is brought to the clinic by her mother with a 2-day history of a rash on her face that
is located under her nasal folds and around her upper lip margin. Her mother states that the rash
began as blisters but has since ruptured, leaving crusting and some sores behind. She reports some
mild itching and discomfort, but otherwise has had no complaints. The patient is otherwise healthy and
takes no medications. She recently started kindergarten. Vaccinations are up to date. Physical
examination reveals multiple erythematous lesions on nasolabial folds bilaterally and along the upper
lip margin. There appears to be some white-brown colored adherent crusting as well. There is no rash
on the palms or soles of the feet. The most likely diagnosis is

A. ecthyma
B. hand-foot-and-mouth disease
C. impetigo
D. herpes simplex type 1
E. erythema infectiosum

181. A 22-year-old gravida 6 para 3-0-2-3 female at 5 weeks' gestation is diagnosed with hyperthyroidism
secondary to Graves disease. Her current and past pregnancies were all unremarkable. You decide to
prescribe her an antithyroid medication. Which of the following medication and trimester pairings is
recommended?

A. propylthiouracil (PTU®) for first, second, and third trimesters


B. propylthiouracil (PTU®) for first trimester and methimazole (Tapazole®) for second and third
trimesters
C. methimazole (Tapazole®) for first trimester and propylthiouracil (PTU®) for second and third
trimesters
D. methimazole (Tapazole®) for first, second, and third trimesters
E. methimazole (Tapazole®) for first and second trimesters and propylthiouracil (PTU®) for third
trimester

75
2018 ACOFP IN-SERVICE EXAMINATION

182. A 75-year-old female presents to the office with the recent onset of mild back pain localized to the mid
thoracic spine. A radiograph of the thoracic spine reveals evidence of a thoracic compression fracture
in the setting of generalized boney demineralization. The most appropriate next step is

A. bone scintigraphy
B. DXA scan
C. bisphosphonate therapy
D. hormone therapy with estrogen
E. calcium replacement therapy

QUESTIONS 183 AND 184 REFER TO THE FOLLOWING INFORMATION


A 33-year-old gravida 1 para 0 female presents to the office for prenatal care. Past medical history is negative
except for receiving a blood transfusion at the age of 16 after a motor vehicle collision while living in Vietnam.
Laboratory studies reveal a positive HBsAg and HBsAb.

183. This patient should be advised regarding which medical therapy upon delivery of her infant?

A. patient must undergo immediate blood transfusion


B. patient will need hepatitis B immune globulin within 12 hours of delivery
C. infant will need hepatitis B immune globulin within 12 hours of delivery
D. patient will need immediate immunization with hepatitis B vaccine
E. infant will need immediate immunization with hepatitis B vaccine

184. Which of the following is correct regarding this patient breast-feeding her infant?

A. unsafe in females with HBsAg


B. safe in females with HBsAg
C. safe only in females with HBsAg who are undergoing treatment for hepatitis B virus infection
D. must pump and feed breast milk via a bottle
E. safe if HBcAb is positive

76

You might also like